ЗАДАЧИ
problems.ru
О проекте | Об авторах | Справочник
Каталог по темам | по источникам |
К задаче N

Проект МЦНМО
при участии
школы 57
Задача 109861
Темы:    [ Линейные рекуррентные соотношения ]
[ Индукция (прочее) ]
Сложность: 4+
Классы: 9,10,11
В корзину
Прислать комментарий

Условие

Автор: Мусин О.

Числовая последовательность a0 , a1 , a2 , такова, что при всех неотрицательных m и n ( m n ) выполняется соотношение

am+n+am-n=(a2m+a2n).

Найдите a1995 , если a1=1 .

Решение

19952 . Полагая m=n , находим a0=0 . Полагая n=0 , получим am+am=(a2m+a0) . Отсюда

Пусть m=n+2 . Тогда a2n+2+a2=(a2n+4+a2n) , и так как в силу 1 a2n+4=4an+2 и a2n=4an , то окончательно получаем:

С другой стороны, в силу 1 и условия a1=1 , имеем:

Сравнивая 2 и 3, заключаем, что последовательность (an) удовлетворяет рекуррентному соотношению

an+2=2an+1-an+2

и начальным условиям a0=0 , a1=1 . Вычислив несколько первых членов последовательности: a2=4 , a3=9 , a4=16 , приходим к предположению, что при всех n0 an=n2 . Доказательство проведем по индукции. При n=0 и n=1 утверждение верно. Пусть оно верно при n=k-1 и n=k ( k1 ). Тогда
ak+1=2ak-ak-1+2= 2k2-(k-1)2+2=(k+1)2,

т.е. утверждение верно и при n=k+1 . Следовательно, a1995=19952 .

Ответ

19952 .

Источники и прецеденты использования

олимпиада
Название Всероссийская олимпиада по математике
год
Год 1995
Этап
Вариант 4
Класс
Класс 11
задача
Номер 95.4.11.6

© 2004-... МЦНМО (о копирайте)
Пишите нам

Проект осуществляется при поддержке Департамента образования г.Москвы и ФЦП "Кадры" .